La ricerca ha trovato 2021 risultati

da Bruno
mar dic 12, 2006 2:28 pm
Forum: Il Forum
Argomento: SPQR (= Son problemi quasi rosicchiati...)
Risposte: 5
Visite : 4080

...

Se tre termini consecutivi della successione:

$\alpha n^{\script 2}+\beta n+\gamma\;$ (con $\;\alpha,\, \beta,\, \gamma, \,n\, \in \, Z$)

sono divisibili per sette, allora anche tutti i
rimanenti termini (prendendo cioè un $\,n\,$
qualsiasi) sono multipli di sette.

Vero o falso?
da Bruno
mar dic 12, 2006 1:49 pm
Forum: Il Forum
Argomento: Contemplazione (ed eventuale calcolo) di un limite
Risposte: 5
Visite : 4198

... Ciao, Gaspero :D Grazie per averci (ri)proposto questo topic, di cui peraltro non mi sono mai accorto. Anche la mia intuizione, di fronte al tuo limite, va per conto suo... e senza mai girarsi! Non lo ricordavo e mi ripiace. Pensa che ancora traballo di bellezza quando guardo l' identità di Eule...
da Bruno
mar dic 12, 2006 12:01 pm
Forum: Il Forum
Argomento: CPME 4-10
Risposte: 2
Visite : 2249

Ciao, Zerinfinito :D
Ho fatto giusto un salto per salutarti!
(Conosco già i problemi e quindi lascio tutto
il divertimento agli altri basecinquini.)

Bruno
da Bruno
mar dic 12, 2006 11:54 am
Forum: Il Forum
Argomento: Molto meno rosicchiato
Risposte: 13
Visite : 8953

Molto meno rosicchiato

... Questo quiz proviene da un altro forum e ha resistito a vari tentativi. A me capitò di trovarne la soluzione non certo in un batter d'occhio, anzi... solo dopo alcune dormite :D Siamo in un supermercato dove tutto ha un prezzo compreso fra 0,99 e 9,99 euro. Un tizio compra tre oggetti, arriva al...
da Bruno
mar dic 12, 2006 11:47 am
Forum: Il Forum
Argomento: SPQR (= Son problemi quasi rosicchiati...)
Risposte: 5
Visite : 4080

Davvero?

... \text Teorema. Siano \small \,a\, e \small \,m\, due numeri reali tali che \small \,a>m . Allora si ha \small \,a=m . \text Dimostrazione. Ponendo \small \,b=\frac{a+m}{2} , si ha: \small \,a+m=2b . Moltiplichiamo ambo i membri per \small \,a-m\, e otteniamo: \small \,a^2-m^2=2ab-2bm , da cui: \...
da Bruno
mar dic 12, 2006 11:46 am
Forum: Il Forum
Argomento: SPQR (= Son problemi quasi rosicchiati...)
Risposte: 5
Visite : 4080

SPQR (= Son problemi quasi rosicchiati...)

... :shock: e :mrgreen: sono due numeri interi positivi (ovvio). Fra le loro potenze seguenti corrono questi fatti: :mrgreen: \,^{\script 19}\, divide esattamente \, :shock: \,^{\script 93} :shock: \,^{\script 19}\, divide esattamente \, :mrgreen: \,^{\script 93} . Bene. Dimostrare che: :mrgreen: \,...
da Bruno
gio dic 07, 2006 2:03 pm
Forum: Il Forum
Argomento: LCPP 13
Risposte: 7
Visite : 4660

...

Naturalmente, (salvo abbagli), i metodi appena visti
si possono estendere anche al caso in cui il quadrilatero
trattato sia più grande di quello formato dai raggi delle
due circonferenze :wink:

PS - Nell'ultima formula, Panurgo, a me verrebbe un coseno
al posto dell'ultimo seno.

Bruno
da Bruno
gio dic 07, 2006 1:57 pm
Forum: Il Forum
Argomento: LCPP 40
Risposte: 12
Visite : 7806

Mhm... senz'altro non ci siamo capiti :D Letto il problema, ho pensato subito a disegnare i due segmenti indicati e cioè a costruirli graficamente, perché questo veniva richiesto. In seguito ho pensato a dimostrare la correttezza della costruzione, com'era doveroso fare. (Non ho postato nulla perché...
da Bruno
gio dic 07, 2006 10:09 am
Forum: Il Forum
Argomento: LCPP 40
Risposte: 12
Visite : 7806

... Matemammamia :mrgreen: Avevo preparato anch'io due scarabocchi ma siete stati troppo veloci (non ho la possibilità di inviare post serali o notturni...) (...) un triangolo giace sul suo lato più lungo (che chiameremo "base")... Naturalmente, la base può anche non essere il lato più lungo del tri...
da Bruno
mar dic 05, 2006 7:05 pm
Forum: Il Forum
Argomento: 4 dimostrazioni di geometria
Risposte: 4
Visite : 4349

Ciao, IlGuista!
Non riesco a fermarmi un minuto di più, purtroppo,
ma intanto prova a dare un'occhiata qui: troverai
sicuramente qualche buono spunto.

Bruno
da Bruno
lun dic 04, 2006 5:50 pm
Forum: Il Forum
Argomento: Indovinello carino....
Risposte: 45
Visite : 30195

eugenio.amitrano ha scritto:CI PROVO, MA PENSO DI SBAGLIARE

1) LOLA - TEO = CUPA
2) LOLA + TEO = 1/CUPA

LOLA = 22, QUINDI

1) CUPA = 22 - TEO
2) 22 + TEO = 1 / (22 - TEO)

484 - TEO^2 = 1
TEO^2 = 483
TEO = +/- SQRT(483)

HAHAHA......
Ma è proprio forte :D
(Malgrado l'indecisione finale...)
da Bruno
lun dic 04, 2006 5:15 pm
Forum: Il Forum
Argomento: LCPP 14
Risposte: 2
Visite : 2515

... Appena il tempo di dire due cose. La prima: bravo Sancho Panza :D La seconda: nel tentativo di rispondere al quesito, dai miei scarabocchi è saltata fuori questa simpatica generalizzazione (che non ricordo di aver visto finora, ma senz'altro dormicchia già da tempo in qualche quaderno, libro o p...
da Bruno
lun dic 04, 2006 12:27 pm
Forum: Il Forum
Argomento: Vera o falsa?
Risposte: 4
Visite : 3275

Bravo Panurgo :D
da Bruno
sab dic 02, 2006 11:12 am
Forum: Il Forum
Argomento: Dal passato
Risposte: 8
Visite : 5688

... Sì, Pasquale, comincio a pensare (meglio tardi che mai...) che tu e Mathmum abbiate ragione. Questi quiz si possono inventare, magari solo con una finalità "contarola", e potrebbero non avere altri significati a monte o a valle o da qualche altra parte... :D Be'... comunque mi è piaciuto pensarc...
da Bruno
ven dic 01, 2006 3:46 pm
Forum: Il Forum
Argomento: Vera o falsa?
Risposte: 4
Visite : 3275

Vera o falsa?

...

Ogni triangolo acutangolo di area $\small \,1\,$è contenuto

in un triangolo rettangolo di area $\small \, \sqr{3}$.

Secondo voi, è vera questa affermazione :?:

(L'ho appena letta ma non ancora ragionata...)



Bruno